Interferencia de luz polarizada

¿Interfiere la luz polarizada?

Sí, lo hace. ¿Podrías ser más específico, por qué crees que no?
La pregunta no está clara pero puede ser útil. No creas que es necesario cerrar, solo vota a favor.
@Kostya: el problema es que, si vota en contra, ¿realmente volverá a no votar si se mejora la pregunta? Creo que es más probable que una pregunta cerrada se "rehabilite"

Respuestas (5)

Hagamos algunos cálculos para no quedar sin fundamento.

1. Polarizaciones perpendiculares.

Primera ola mi 1 X = mi 0 porque ω t , Segunda ola mi 2 y = mi 0 porque ( ω t + Δ ) . Aquí Δ es una diferencia de fase entre las ondas.

Campo total:

mi = mi 0 ( i porque ω t + j porque ( ω t + Δ ) ) .

Intensidad:

yo | mi | 2 = mi 0 2 porque 2 ω t + porque 2 ( ω t + Δ ) ,

donde el promedio se define como F ( t ) = 1 T 0 T d t F ( t ) (de modo que porque 2 ( ω t + Δ ) = 1 2 para cualquier Δ ).

Finalmente tenemos yo mi 0 2 , que es independiente de la diferencia de fase entre las ondas.

2. Polarizaciones paralelas.

Primera ola mi 1 X = mi 0 porque ω t ,Segunda ola mi 2 X = mi 0 porque ( ω t + Δ ) .

Campo total:

mi = i mi 0 ( porque ω t + porque ( ω t + Δ ) ) .

Intensidad:

yo mi 0 2 porque 2 ω t + 2 porque ω t porque ( ω t + Δ ) + porque 2 ( ω t + Δ ) = mi 0 2 ( 1 + porque Δ ) ,
que muy bien depende del cambio de fase entre las ondas.

Buena manera de explicar esto, definitivamente más completa que mi respuesta.
De acuerdo, +1 por contenido y estilo. (pequeño error tipográfico en la primera línea, sin embargo: "no debe ser sin fundamento")
¡Gracias a todos por sus comentarios! Por mi respuesta, debe considerarse como una continuación de @Colin K.
Kostya, cuando sumas dos campos perpendiculares, ¿piensas que consideras una interferencia? ¡Cualquier interferencia está hecha por definición de la misma polarización! ¡Considera un interferómetro de un brazo!
Eso es lo que dijo, y también explicó por qué es así.
Le daría +1 si enfatizara el requisito de coherencia y la interferencia de la misma longitud de onda, solo que un poco más obvio. (Aunque las fórmulas implican eso)
En mi opinión, hay un error en el último eq (intensidad de los componentes paralelos): debe dar 0 cuando delta = pi -- compruebe que lhs != rhs wolframalpha.com/input/?i=(cos(t)) ^2+ %2B2*cos(t)*cos(t%2Bdelta)%2B(cos(t%2Bdelta))^2+-+(1-cos(delta)) -- es el rhs el que contiene el error.
el editor utiliza símbolos adicionales que impiden la copia correcta del enlace, lo siento
¿Está bien la derecha de la última ecuación?.
¿Has notado el promedio sobre t?
Delta=pi, w=1,E0=1 entonces E(t)=cos(t)+cos(t+pi)=0 y |E(t)|^2 = 0 <> 2, Si E(t) = 0 para cualquier t entonces el promedio es 0.
Sí, tiene usted razón. Perdí el signo, debe ser "+" en lugar de "-".

Sí. De hecho, la luz solo interferirá con la luz de la misma polarización. Si toma un interferómetro de Mach-Zehnder , por ejemplo, y coloca una óptica giratoria de polarización (una placa de ondas) en uno de los brazos, el patrón de interferencia perderá contraste. Si la polarización se gira 90 grados, el patrón desaparecerá por completo.

Sólo puedo añadir que mecánicamente cuánticamente un fotón interfiere consigo mismo, es decir, el campo resultante es siempre de la misma polarización. Si interviene con la óptica de rotación de polarización, infringe la regla principal: no intervenir en los brazos del interferómetro. Cualquier intervención de este tipo (polarización, intensidad, etc.) estropea la charla en tal o cual medida.
debe mencionar algo como coherencia y solo interferencia de la misma longitud de onda

Como han señalado otros, no obtendrá ninguna modulación de intensidad de la interferencia de dos haces de luz polarizados linealmente con polarizaciones ortogonales. Sin embargo, vale la pena señalar que esto no significa que los haces con polarizaciones perpendiculares no se afecten entre sí. De hecho, un par de haces de contrapropagación con polarizaciones lineales ortogonales, la llamada configuración "lin-perp-lin", es el mejor sistema para comprender el efecto de enfriamiento de Sísifo, cuya explicación fue una gran parte de el Premio Nobel de Física de 1997 .

La superposición de dos haces polarizados linealmente que se propagan en sentido contrario con polarizaciones ortogonales no proporciona ninguna modulación de intensidad, pero crea un gradiente de polarización. Para la configuración lin-perp-lin, obtiene regiones alternas de polarización circular izquierda y derecha, y combinada con el bombeo óptico, esto le permite configurar un escenario en el que puede enfriar los vapores atómicos a temperaturas extremadamente bajas. Esto hace que el enfriamiento por láser sea mucho más útil de lo que sería de otra manera y permite todo tipo de tecnologías geniales como los relojes de fuentes atómicas.

No es una interferencia en el sentido en que generalmente se entiende, pero es un fenómeno genial que resulta de la superposición de haces con diferentes polarizaciones. Por lo tanto, no debe pensar que solo porque no produce un patrón de puntos brillantes y oscuros no es interesante.

Su pregunta es bastante vaga, pero en resumen, la respuesta es: , búsquelo en wikipedia . Pero seamos más precisos:

Siempre que la intensidad de la luz sea lo suficientemente baja como para no obtener efectos no lineales , el principio de superposición de la óptica lineal es válido. Eso significa que las amplitudes de dos campos electromagnéticos (EM) se suman, lo que produce interferencia.

Sin embargo, dado que las amplitudes son vectores (mientras que la intensidad, al estar relacionada con los cuadrados absolutos de las amplitudes, es un escalar), la interferencia depende de la polarización relativa , la intensidad total para dos ondas EM polarizadas linealmente es

yo = yo 1 + yo 2 + 2 yo 1 yo 2 porque ( Δ φ )

dónde Δ φ denota el ángulo entre las dos polarizaciones. Verá que para la polarización perpendicular el término del coseno desaparece, las intensidades simplemente se suman y no obtiene interferencia, mientras que para la polarización antiparalela ( Δ φ = 180 ° ) obtienes interferencia destructiva ya que el coseno se convierte en -1. En caso de que se pregunte acerca de la conservación de la energía (que sea proporcional a la intensidad), tenga en cuenta que solo se conserva la energía global, mientras que las fluctuaciones locales están bien.

Una nota final: todo solo funciona para una relación de fase bien definida entre dos ondas EM. Es decir, solo los componentes espectrales de la misma longitud de onda pueden interferir, y la longitud y el tiempo de coherencia deben ser lo suficientemente grandes; no obtendrá una interferencia perfecta si su fuente de luz parpadea debido al calor, por ejemplo.

Sí, lo hace y esta propiedad es independiente de una polarización particular. Entonces, la luz no polarizada da el mismo patrón de interferencia.

-2 para que? ¿No respondí la pregunta?
porque está mal o al menos es muy engañoso. Si bien afirmar que "siempre ocurre una interferencia" es válido a pesar de omitir la dependencia de la coherencia y la diferencia de fase, su declaración non polarized light gives the **same** interference patternes realmente incorrecta
Ah, sí, insinué la misma frecuencia, por supuesto, y las condiciones del interferómetro. Es especialmente evidente cuando hablo de la interferencia del fotón consigo mismo.